Search found 6 matches

Return to advanced search

by hakopis
Wed Jun 11, 2014 4:53 pm
 
Forum: Section #1
Topic: Q18 - Some planning committee members
Replies: 19
Views: 6397
Jump to post

Re: Q18 - Some planning committee members

Wasn't sure about the answer at first, BUT I found a logical map once I saw it was (E). Setup: PC --(some)-- SFI or A --(some)--B SFI -------> RCI or B -------> C PC -------> ~LS or A -------> ~D PC --(many)-- WS or A --(many)--E Where: PC = Planning committee members SFI = Significant F...
by hakopis
Wed Nov 05, 2014 12:58 pm
 
Forum: Section #3
Topic: Q20 - Whoever murdered Jansen was undoubtedly
Replies: 19
Views: 6698
Jump to post

Re: Q20 - Whoever murdered Jansen was

This argument concludes that Samantha must be the killer. For this to be true, we could plug in a variety of things to force this. The sufficient assumption in this case was that those two individuals were the only individuals in the room. This forces Samantha to be the killer since Herbert is logi...
by hakopis
Wed Nov 05, 2014 3:03 pm
 
Forum: Section #2
Topic: Q19 - Vanwilligan: Some have argued
Replies: 4
Views: 3406
Jump to post

Re: Q19 - Vanwilligan: Some have argued that

Is there any tips on how to eliminate answer choices that are the reverse of what we want? Like (E) vs. (D). in this problem. I usually get it down to the right answer and the reverse but have a hard time eliminating the reverse. Thanks! I'm not sure whether you still care, but... The way I ended u...
by hakopis
Sat Nov 08, 2014 4:38 pm
 
Forum: Section #1
Topic: Q12 - Poor nutrition is at the root
Replies: 22
Views: 9557
Jump to post

Re: Q12 - Poor nutrition is at the root

@timsportschuetz: However, if the causal statement appears in the CONCLUSION, then we ALWAYS infer that the causal statement is LOGICALLY INVALID! WaltGrace1983: My Thought Process: Here we go again! Correlation and Causation. This argument is saying that, after given some good food for four months...
by hakopis
Sun Nov 09, 2014 6:26 pm
 
Forum: Section #4
Topic: Q1 - A government ought to
Replies: 4
Views: 2374
Jump to post

Re: Q1 - A government ought to

So, if I were to put in sentential logic form, would it look like this:

Premise: HN --> FS

Intermediary: FS --> BI

Missing Ass: G --> BI

Conclusion: G --> FS
by hakopis
Sun Nov 09, 2014 9:03 pm
 
Forum: General Questions - LR
Topic: Negation for Necessary Assumption Questions
Replies: 8
Views: 3024
Jump to post

Re: Negation for Necessary Assumption Questions

Are there any reasonable differences between 1) Arguments that depend/require etc. and 2) Conclusions that depend/require etc.? If so, what changes/modifications should be made to our strategy? As of now, I weed out 3/5 wrong answer through POE and use the negation technique for the last two. I also...